A delivery person uses a service elevator to bring boxes of books up to an office. The delivery person weighs 160 lb and each box of books weighs 40 lb. The maximum capacity of the elevator is 670 lb. How many boxes of books can the delivery person bring up at one​ time?

Answers

Answer 1

Answer:

Step-by-step explanation:

So if the delivery person weights 160 lbs than we’re going to subtract 670-160=510 next we divide 40 into 510 which is 12.75. Which means that he can bring 12 boxes.

Answer 2

Answer:

12

Step-by-step explanation:

max cap 670lb- delivery person 160lb  weight left 510lb

weight 510lb/each box 40lb number of boxes 12.75lb

each box 40lb times number of boxes 12lb  480lb+160lb 640lb

so your still have 30 lb before max weight of elevator


Related Questions

Hi May I know how to solve this?

Answers

Answer:

k = 3/4

Step-by-step explanation:

Parallel lines have the same slope so first, we need to find the slope of 8x + 6 = 2y. To do so, we can do the following:

8x + 6 = 2y

2y = 8x + 6

y = 4x + 3 (This is now in slope intercept form - y = mx + b where m = slope and b = y-int.)

Therefore, the slope is 4. Now, we can do:

3x = ky - 10

3x - 10 = ky

(3x / k) - (10 / k) = y

y = (3x / k) - (10 / k)  (where k ≠ 0)

Now that the second line is in slope-intercept form, we can set the two x terms equal to each other, therefore:

3x / k = 4x

3x = 4x * k

3x / 4x = k

k = 3/4

Solving Quadratics



Solve for all values of x by factoring. x² + 4x + 5 = -6x + 5​

Answers

Answer: x=5 and x=-1

Step-by-step explanation:

X^2-4x-5=0

(X-5) (x+1)=0

X-5=0. And. X+1=0

(4x5 - 13x2 - 10) - (2x5 - 9x2 + 15)


2x5 - 11x2 + 5

2x5 - 4x2 - 25

2x5 - 4x2 + 5

-27x7

Next QuestionNext

Answers

Answer:

2x5-4x2-25

Step-by-step explanation:

First you distribute the negative sign to the second equation which makes each number it's opposite

Which would result in 4x5-13x2-10-2x5+9x2-15

Then you combine like terms and get 2x5-4x2-25

Answer:

-23

Step-by-step explanation:

A recipe that makes 25 oatmeal cookies calls for 2.5 cups of oats and one cup of sugar. Jerry needs to make 195 cookies for his party. How many cups of oats will he need?

Answers

you would need 19.5 cups of oats and 7.8 cups of sugar because 195 divided by 25 is 7.8 so then you just multiply the both of them by that

There are nine teddy bears in the diagram below. How many TOTAL squares are
there?

Answers

Answer: 14

Step-by-step explanation: 9 small squares with one teddy bear in each, 4 medium squares with 4 bears in each medium square, and 1 big square for the entire thing.

The total number of square is 14

What is figure counting?

In the counting of figures, you have a shape or a figure. From the given shape you will have to identify a given known shape and count the number of times it is present in the given shape.

According to the question

9 small squares with one teddy bear in each,

4 medium squares with 4 bears in each medium square.

1 big square for the entire thing.

Total square = 9 + 4 + 1

                     = 14

Hence, the total number of square is 14

To learn more about counting figure from here

https://brainly.com/question/17456957

#SPJ2

Brandon wants to buy a new bike. The model he likes is on sale for 15% off its original price. His parents agree to pay for $50 of the cost. If the original cost of the bike is x dollars, which of the expressions below represent the amount Brandon has to pay?

Answers

Answer:

0.85x - $50

Step-by-step explanation:

Given that :

Brandon wants to buy a new bike. The model he likes is on sale for 15% off its original price. His parents agree to pay for $50 of the cost. If the original cost of the bike is x dollars, which of the expressions below represent the amount Brandon has to pay

Discount on price = 15%

Amount parents agreed to pay = $50

Cost of bike = x

Discount on bike = 0.15x

Cost after discount is removed = (x - 0.15x) = 0.85x

Parent pays $50 of cost price, amount Brandon will have to pay equals :

(0.85x - $50)

Answer:

0.85x - $50

Step-by-step explanation:

add r to q, then add s to the result

Answers

Answer:

(r+q)+s

Step-by-step explanation:

The parentheses make it clear that r and q need to be added first.

On James’s MP3 player, he has 12 sad songs and 40 upbeat songs that he wants to put into playlists. He wants to have the same number of sad songs and upbeat songs in each playlist. What is the maximum number of playlists that he can create?

Answers

Answer:

4 playlists

:)

A large western state consists of 3129 million acres of land. Approximately 55% of this land is federally owned. Find the number of acres that are not federally owned

Answers

Answer:

1408.5 acres

Step-by-step explanation:

If 55% of the land is federally owned, then 100 - 55 = 45% of the land is not federally owned.

We need to find how many acres are not federally owned, in other words, we need to find how many acres represent 45% of the total 3129 acres. So, to do this we can apply a rule of three:

3129 acres ---- 100%

x acres     ------ 45%

[tex]x=\frac{(3129)(45)}{100} =3129(0.45)= 1408.05[/tex]

Therefore, the total of acres that are not federally owned are 1408.5 acres.

Given x=-3, y=6, and z=-4 x+y+(-1)=

Answers

Answer:

8

Step-by-step explanation:

x=3

y=6

to find x+y-1, sub in x and y

3+6-1 = 8

Answer: 8

Step-by-step explanation:

x+y+(-1), given x=3, y=6, and z=-4

---------

 x+y+(-1)

=(3)+(6)+(-1) ⇔ substitute x and y

=9+(-1) ⇔ add 3 and 6

=8

Hope this helps!! :)

Please let me know if you have any questions

8 gal to 9 1/3 gal write ratio in lowest form with whole numbers in numerator and denominator

Answers

Answer:

[tex]\bold{\dfrac{6}{7}}[/tex]

Step-by-step explanation:

Given ratio is:

8 gal to [tex]9\frac{1}3[/tex] gal

To find:

Ratio in Lowest form with whole numbers in numerator and denominator.

Solution:

First of all, let us convert [tex]9\frac{1}3[/tex] to simple [tex]\frac{p}{q}[/tex] form:

Formula:

[tex]a\dfrac{b}{c} = \dfrac{a\times c+b}c[/tex]

Using the formula, we get:

[tex]9\dfrac{1}{3} = \dfrac{9\times 3+1}{3} = \dfrac{28}{3}[/tex]

Now, let us have a look at the ratio:

[tex]8 : \dfrac{28}{3}[/tex]

Any ratio, [tex]p:q[/tex] can be written as [tex]\frac{p}{q}[/tex].

Writing given ratio as per above:

[tex]\dfrac{8}{\dfrac{28}{3}}\\\Rightarrow \dfrac{8 \times 3}{28}\\\Rightarrow \dfrac{2\times 3}{7}\\\Rightarrow \bold{\dfrac{6}{7}}[/tex]

Therefore, the answer is

[tex]\bold{\dfrac{6}{7}}[/tex]

Use the Distance Formula to answer the following questions. Round all of your answers to the nearest tenth (one decimal place). 2) Find the distance between the points (-3, 1) and (5,-2). Round your answer to the nearest teeth. ​

Answers

Answer:

distance= 9 units to the nearest

Step-by-step explanation:

using the formula

√(∆x)^2+(∆y)^2

√73

= 8.5

= 9 units

Answer:  8.5 units approximately

===========================================

Work Shown:

d = distance between (-3,1) and (5,-2)

[tex]d = \sqrt{(x_1 - x_2)^2 + (y_1 - y_2)^2}\\\\d = \sqrt{(-3-5)^2 + (1-(-2))^2}\\\\d = \sqrt{(-3-5)^2 + (1+2)^2}\\\\d = \sqrt{(-8)^2 + (3)^2}\\\\d = \sqrt{64 + 9}\\\\d = \sqrt{73}\\\\d \approx 8.5440037\\\\d \approx 8.5\\\\[/tex]

Alternatively, you can plot the two points on the same xy grid. Then form a right triangle with the two points as the endpoints of the hypotenuse. Use of the pythagorean theorem will result in getting the same approximate distance. The distance formula is effectively the same as the pythagorean theorem, but just in a different form.

Name the Reflection Rule that results in the following transformation

A(1,4), B(2, 7), C(9, 3) to

A'(1,-4), B'(2, -7), C'(9, -3)

Answers

Reflect across the x-axis because the rule is (x, -y)

someone tell me the answers tho these please. Part A: Factor 3x2y2 − 2xy2 − 8y2. Show your work. (4 points) Part B: Factor x2 + 10x + 25. Show your work. (3 points) Part C: Factor x2 − 36. Show your work. (3 points)

Answers

Step-by-step explanation:

x2 + 10x + 25

x2 + 5x + 5x + 25

x(x + 5) + 5(x + 5)

(x + 5)(x+ 5) are the factors

x2 - 36

Both are squares so

x2 - (4)2

5. Which of the following is a multiple of 8?
A 4
C 20
B 8
D 100

Answers

Answer:

8

Step-by-step explanation:

8 can be divisible by 8. Therefore, it would be a multiple of 8.

The first multiple of a number is always that number.

All other options given are not multiples of 8.

Hope this helps.

B) 8 since 8 is one of the multiple of 8 and it’s the beginning of the multiples of 8


Solve -4(x + 1) – 3 = -3(x - 4).

Answers

Answer:

x =-19

Step-by-step explanation:

[tex]-4(x + 1) - 3 = -3(x - 4).\\\\\mathrm{Expand\:}-4\left(x+1\right)-3:\quad -4x-7\\\\\mathrm{Expand\:}-3\left(x-4\right):\quad -3x+12\\-4x-7=-3x+12\\\\\mathrm{Add\:}7\mathrm{\:to\:both\:sides}\\-4x-7+7=-3x+12+7\\\\Simplify\\-4x=-3x+19\\\\\mathrm{Add\:}3x\mathrm{\:to\:both\:sides}\\-4x+3x=-3x+19+3x\\\\Simplify\\-x=19\\\\\mathrm{Divide\:both\:sides\:by\:}-1\\\frac{-x}{-1}=\frac{19}{-1}\\\\Simplify\\x=-19[/tex]

Answer:

We have this equation:

-4(x+1) - 3 = -3(x-4)

First of all, we solve the paretheses:

-4x - 4*1 - 3 = -3x - (-3*4)

-4x - 4 - 3 = -3x +  12

-4x -7 = -3x + 12

We sum 7 on both sides and 3x on both too:

-4x - 7 + 7 + 3x = -3x + 12 + 3x + 7

-4x + 3x = 12 + 7

-x = 19

x = -19

Now we can verify:

-4(-19+1) - 3 = -3(-19-4);

-4(-18) - 3 = -3(-23);

72 - 3 = 69 (TRUE)

3(x + 5) = 2(4x + 9) ​

Answers

Answer:

x=-3/5

Step-by-step explanation:

3(x + 5) = 2(4x + 9) ​

First, distribute:

3(x)+3(5)=2(4x)+2(9)

3x+15=8x+18

-5x= 3

x= -3/5

Which of the following options correctly represents the complete factored form of the polynomial?

Answers

Answer:

B

Step-by-step explanation:

f(x)=(x-3)(x²+2x+2)

when solve for x using quadratic formula fo x in (x²+2x+2):

x=(-b±√b²-4ac)/2a  a=1,b=2,c=2

x=-1±i

f(x)=(x-3)(x+1-i)(x+1+i)

The complete factored form of the polynomial is F(x) = (x - 3)(x + 1 + i)(x + 1 - i).

To find the complete factored form of the polynomial F(x) = x³ - x² - 4x - 6, we can factor it using various methods such as synthetic division or factoring by grouping.

Factoring the polynomial, we get:

F(x) = (x + 3)(x - 1)(x + 2)

Comparing the factored form with the options provided:

A. F(x) = (x + 3)(x + 1)(x - 1)

This option does not match the factored form of the polynomial.

B. F(x) = (x - 3)(x + 1 + i)(x + 1 - i)

f(x)=(x-3)(x²+2x+2)

When solve for x using quadratic formula fo x in (x²+2x+2):

x=(-b±√b²-4ac)/2a  

x=-1±i

f(x)=(x-3)(x+1-i)(x+1+i)

So, the is the required factored form.

C. F(x) = (x - 3)(x + 1 + i)(x - 1 - i)

This option does not match the factored form of the polynomial.

D. F(x) = (x + 3)(x + 1 + i)(x + 1 - i)

This option does not match the factored form of the polynomial.

Learn more about Factor here:

https://brainly.com/question/15872577

#SPJ6

please answer and give reasoning. might give brainliest.

Answers

I think the answer is A but don’t count me on it

Answer:

A

Step-by-step explanation:

A contains √3 --> irrational number because 3 is not a square number

For other choices:

- All numbers in the form a/b (a,b are integer numbers) are rational numbers

- √36, √9, √4, √25, √49, √100 are integer numbers because (36, 9, 4,....) are square numbers

- All given decimal numbers can be written in the fraction form

0.5 = 1/2

39.77 = 39 + 77/100 = 3977/100

0.15151578 and 0.00010001 are finite decimals, 0.303030... is infinite repeating decimal, so technically they are rational numbers.


solve for x and y please

Answers

Answer:

Y=40

X=67

Step-by-step explanation:

In a cross line problem, one part of the cross, (such as 63°) is always the same on the other side. (like y+23° and 63°. This means Y=40 because 40+23 is 63.

To find x, you must know that 360° is all the way around in degrees. you know that 126° (63 +63) is already taken up. So you subtract 126 from 360 and then multiply the answer by 1/2 because you trying to find the bottom total.(360-126)=234(1/2)= 117 Lastly, you find X in the equation 2x-17=117.

x=67

A rectangular pool is twice as
long as it is wide. What are the
dimensions of the pool if the
perimeter is 48 yd?

Answers

Step-by-step explanation:

L=2B

P=2(L+B)

48=2(2B+B)

48=2(3B)

48=6B

B=8yd

L=2*8

=16yd

Answer:

length * width = 16 yd * 8 yd

or

length = 16 yd

width = 8 yd

Step-by-step explanation:

take the rectangular pool to have sides a (length) and 2a (width).

we are told that the length is as twice as its width, therefore,

Perimeter is given by (a + 2a) × 2 = 48 yd

Divide both sides by 2 ⇒ [tex]\frac{((a + 2a) * 2)}{2} = \frac{48 yd}{2}[/tex] ⇒ a + 2a = 24 yd

⇒3a = 24 yd

Divide both sides by 3 ⇒ [tex]\frac{3a}{3} = \frac{24 yd}{3}[/tex] ⇒ a = 8 yd

therefore, the width of the pool is 8 yd and its length is 16 yd (Twice as long as it is wide)

Can someone help me with this problem please??? If ƒ( x ) = x 2 + 1 and g( x ) = 3 x + 1, find 2 · ƒ(4). A. 18 B.34 C.65

Answers

Answer:  B  

Step-by-step explanation:

They've given you two functions  f(x) = x^2 + 1 and g(x) = 3x +1  And they are you to find 2 * f(4) then g(x) is not needed so input in  4 for the function f(x) and multiply the output by 2.

F(4) = 4^2 + 1

F(4) = 16 +1

f(4) =  17  

   17  * 2 = 34

Answer:

B.34

Step-by-step explanation:

[tex]f( x ) = x^ 2 + 1 \\ g( x ) = 3 x + 1\\\\f(4) \times 2 = ?\\f(4) = 4^2 +1\\\\=( 16+1) \times 2\\\\= 17\times 2\\\\= 34[/tex]

What is the area of 6ft and 5ft

Answers

Answer:

30 ft

Step-by-step explanation:

6 x 5 = 30

you multiply 6 times 5 and you get 30

Step-by-step explanation:

AB is a straight line

Work out the size of angle x

Answers

Wouldn’t you just subtract the 132 degrees from 180 degrees ?

20 POINTS!!! NEED ALL 4 ANSWERED!!!

Set the two sides equal to each other and solve for x!

① -2x + 81 = 61

② 9x - 58 = 5

③ 21 + 10x = 3x + 28

④ 9x - 42 = 2x

(I already put them in the correct forms based on each images But here's the images anyways, just in case.)

Answers

Answer:

1. x = 10  2. x = 7  3. x = 1  4. x = 6

Explanation:

1. -2x + 81 = 61 (Subtract 81 from 61)

  -2x = -20 (Divide)

   x = 10

2. 9x - 58 = 5 (Add 58 to 5)

   9x  = 63 (Divide)

   x = 7

3. 21 +10x = 3x + 28 (Rearrange expression)

   21 - 28 = 3x - 10x (Combine like terms)

   -7 = -7x (Divide)

    x = 1

4. 9x - 42 = 2x (Rearrange)

   9x - 2x = 42 (Combine like terms)

   7x = 42 (Divide)

   x = 6

Find the sum: 3.85 + 6.29 =

Answers

Answer:

3.85 + 6.29 = 10.14

Hoped I helped

Step-by-step explanation:

3.85+6.29= 10.14

thank

The function h(x)=4/3x-1 represents the composite h(x)=f(g(x)).if f(x)=2x-1,what is g(x)

Answers

Answer:

[tex]g( x) = \frac{2}{3}x[/tex]

Step-by-step explanation:

Given:

Composite Function:

[tex]h( x)=f( g( x))[/tex]

Also,

[tex]h( x)=\frac{4}{3}x-1[/tex]

To find:

[tex]g( x ) = ?[/tex]

Solution:

First of all, let us learn about a composite function.

Composite function [tex]f( g( x))[/tex] means to write [tex]g( x)[/tex] in place of [tex]x[/tex] in the function [tex]f( x)[/tex].

Let [tex]g( x) = y[/tex]

So, [tex]f( g( x))[/tex] =  [tex]f( y )[/tex].

Therefore,

[tex]h( x) =f( g( x)) = f( y)[/tex]

So, [tex]f( y) = 2y-1[/tex]

We have to solve for [tex]y[/tex] in terms of [tex]x[/tex] to find the value of [tex]g(x)[/tex]:

[tex]\Rightarrow \frac{4}{3}x-1 = 2y-1\\\Rightarrow 2y=\frac{4}{3}x\\\Rightarrow y = \frac{2}{3}x[/tex]

Hence, the answer is [tex]g( x ) = \frac{2}{3}x[/tex].

Checking whether the answer is correct or not:

[tex]f( g( x)) = f( \frac{2}{3}x) = 2\times \frac{2}{3}x -1 = \frac{4}{3}x-1[/tex]

which is [tex]h( x)[/tex].

Hence, our answer [tex]g( x ) = \frac{2}{3}x[/tex] is correct.

The g(x) of the function is:

g(x) = (2/3)x

How to find what g(x) is?

A function is a relationship between inputs where each input is related to exactly one output.

Every function has a domain and codomain or range. A function is generally denoted by f(x) where x is the input.

We have:

h(x) = (4/3)x - 1

h(x) = f(g(x))

f(x) = 2x - 1.

Thus, we can say:

h(x) = f(g(x)) = 2*g(x) - 1

(4/3)x - 1 = 2g(x) - 1

(4/3)x - 1 + 1= 2g(x)

(4/3)x = 2g(x)

(4/3)x * 1/2 = g(x)

(2/3)x = g(x)

g(x) = (2/3)x

Therefore, g(x) = (2/3)x.

Learn more about function on:

brainly.com/question/29233399

#SPJ6

Evaluate the expression: 6:3+2x7

Answers

Answer:

15.41

Step-by-step explanation:

i need help with this page​

Answers

Answer:

<abc=14

Step-by-step explanation:

<abc= 14 ; because if the whole thing is 90 and <CBD is already 76 then you subtract 90-76 to get the rest of the angle which is 14 degrees

the first half of food+the last quarter of door​

Answers

Answer:

For

Step-by-step explanation:

the first half of food + the last quarter of door

= F o o d + d o o r

= Fo + r

= For

Other Questions
Which of the following types of energy does an object store in the bonds of atoms and molecules? (5 points)Chemical energyObGravitational energyKinetic energyOdSound energy Why might someone want to implicate you with a certain group? QUICKKKK I NEED HELP FASTT ........ Ford and General Motors are considering expanding into the Vietnamese automobile market. Devise a simple prisoners' dilemma game to demonstrate the strategic considerations that are relevant to this decision. You and your firm need to develop policies that avoid acquisitions and restructuring. Select one: True False Farmer Ali had a well on his property 200 feet deep. When the well went dry, he hired a company to drill the well deeper. He used the equation d = -75t 200 to find the depth of the well, d, using the number of days the company drilled, t. What is the slope of the line? Why the partition of Bengal is considered as the pivotal point in the formation of the Pakistan in 1947? 5 marks question plz answer fast 140 + 150 + 160 + 180 + 190 + 200 + 210 Arithmetic Sequence Tara is mowing her lawn. She mows 3/5 of her 925-square meter lawn before stopping to chatwith a neighbor. How many square meters does Tara have left to mow after chatting with herneighbor? Create a simplified list of Machiavelli's Rules for Rulers based on hisadvice in The Prince. Restaurants often slip takeout menus under Braden's apartment door. The table shows how many menus Braden has collected from different types of restaurants. Japanese 24 Chinese 1 Thai 9 Indian 4 What is the probability that one of Braden's takeout menus, selected at random, will be from an Indian restaurant? Write your answer as a fraction or whole number. Cristina, yo voy al cine, quieres venir _______? A) conmigo Ral, t puedes llevar a mi mam ________ al aeropuerto el sbado? B) con ellos Seor y seora Guzmn, sus hijos navegan ________ en el crucero. C) con ustedes Son Nico y Tomasa. Su hija jug ________ en el aeropuerto mientras esperaban. D) contigo I have to find an equation for a table Use evidence from the passage to write a paragraph explaining how Galarza uses DESCRIPTIVE language to bring his narrative to life? what is the farthest thing in space we know about? what is this help -11 =-8x What does the word fixed mean in the stage direction trifles Susan glaspell. Eyes fixed on a loaf of bread.. How is sound produced in our throat If the demand curve for a life-saving medicine is perfectly inelastic, then a reduction in supply will cause the equilibrium price to: Evaluate 13+6/y when y=6 Imagine you are writing a formal argument to present, could be to your parents,peers or professionals, what would you write an argument for and why?PLEASE GIVE ME AN IDEA on WHAT KIND OF ARGUMENT I SHOULD DO!!!